Solving an Intriguing Integral: x-1 in the First Quadrant

  • Thread starter Thread starter Knissp
  • Start date Start date
  • Tags Tags
    Integral
Click For Summary
The integral to solve is ∫∫_R (x-1) dA, where R is the region in the first quadrant between y=x and y=x^3. The bounds for integration were set as ∫ from x=0 to 1 and y=x^3 to y=x. The calculated result is -7/60, which was verified using a computer algebra system (CAS). There is a discrepancy with the textbook answer of -1/2, raising the possibility of a misprint or an error in the bounds. The setup appears correct based on the provided region.
Knissp
Messages
72
Reaction score
0

Homework Statement


\int\int_R x-1 dA
R is the region in the first quadrant enclosed between y=x and y=x^3



Homework Equations





The Attempt at a Solution



I set up the bounds as follows: \int_{x=0}^1\int_{y=x^3}^x x-1 dydx

Integrating, I get -7/60, verified with CAS.

I thought this was an easy problem but my answer doesn't match the textbook (-1/2 but could be a misprint, right?) or did I somehow put the bounds of integration wrong?
 
Physics news on Phys.org
It looks fine to me.
 
I agree with your answer
 
Question: A clock's minute hand has length 4 and its hour hand has length 3. What is the distance between the tips at the moment when it is increasing most rapidly?(Putnam Exam Question) Answer: Making assumption that both the hands moves at constant angular velocities, the answer is ## \sqrt{7} .## But don't you think this assumption is somewhat doubtful and wrong?

Similar threads

  • · Replies 19 ·
Replies
19
Views
2K
Replies
9
Views
2K
Replies
3
Views
2K
Replies
4
Views
2K
  • · Replies 2 ·
Replies
2
Views
2K
  • · Replies 4 ·
Replies
4
Views
2K
Replies
2
Views
1K
  • · Replies 10 ·
Replies
10
Views
2K
Replies
29
Views
4K
  • · Replies 7 ·
Replies
7
Views
2K